General derivative of the exponential operator w.r.t. a parameter












5












$begingroup$


I am interested in the calculation of the general $N$th derivative w.r.t. a parameter $lambda$ of a quantum mechanical exponential operator with the following structure:
begin{equation*}
frac{mathrm d^N}{mathrm d lambda^N} e^{-beta hat H(lambda)}
end{equation*}

In the case $n=1$ this reads as
begin{equation*}
sum_{n=0}^{+infty}sum_{m=0}^{n-1}frac{1}{n!}[hat H(lambda)]^{m}frac{mathrm dhat H(lambda)}{mathrm d lambda}[hat H(lambda)]^{n-m-1}
end{equation*}

Is there a compact fashion to rearrange the expression above for the general $N$th derivative?










share|cite|improve this question











$endgroup$








  • 1




    $begingroup$
    WP.
    $endgroup$
    – Cosmas Zachos
    Feb 20 at 12:15
















5












$begingroup$


I am interested in the calculation of the general $N$th derivative w.r.t. a parameter $lambda$ of a quantum mechanical exponential operator with the following structure:
begin{equation*}
frac{mathrm d^N}{mathrm d lambda^N} e^{-beta hat H(lambda)}
end{equation*}

In the case $n=1$ this reads as
begin{equation*}
sum_{n=0}^{+infty}sum_{m=0}^{n-1}frac{1}{n!}[hat H(lambda)]^{m}frac{mathrm dhat H(lambda)}{mathrm d lambda}[hat H(lambda)]^{n-m-1}
end{equation*}

Is there a compact fashion to rearrange the expression above for the general $N$th derivative?










share|cite|improve this question











$endgroup$








  • 1




    $begingroup$
    WP.
    $endgroup$
    – Cosmas Zachos
    Feb 20 at 12:15














5












5








5


0



$begingroup$


I am interested in the calculation of the general $N$th derivative w.r.t. a parameter $lambda$ of a quantum mechanical exponential operator with the following structure:
begin{equation*}
frac{mathrm d^N}{mathrm d lambda^N} e^{-beta hat H(lambda)}
end{equation*}

In the case $n=1$ this reads as
begin{equation*}
sum_{n=0}^{+infty}sum_{m=0}^{n-1}frac{1}{n!}[hat H(lambda)]^{m}frac{mathrm dhat H(lambda)}{mathrm d lambda}[hat H(lambda)]^{n-m-1}
end{equation*}

Is there a compact fashion to rearrange the expression above for the general $N$th derivative?










share|cite|improve this question











$endgroup$




I am interested in the calculation of the general $N$th derivative w.r.t. a parameter $lambda$ of a quantum mechanical exponential operator with the following structure:
begin{equation*}
frac{mathrm d^N}{mathrm d lambda^N} e^{-beta hat H(lambda)}
end{equation*}

In the case $n=1$ this reads as
begin{equation*}
sum_{n=0}^{+infty}sum_{m=0}^{n-1}frac{1}{n!}[hat H(lambda)]^{m}frac{mathrm dhat H(lambda)}{mathrm d lambda}[hat H(lambda)]^{n-m-1}
end{equation*}

Is there a compact fashion to rearrange the expression above for the general $N$th derivative?







quantum-mechanics operators mathematical-physics hamiltonian differentiation






share|cite|improve this question















share|cite|improve this question













share|cite|improve this question




share|cite|improve this question








edited Feb 20 at 11:17









Qmechanic

105k121901203




105k121901203










asked Feb 20 at 9:54









GrazGraz

505




505








  • 1




    $begingroup$
    WP.
    $endgroup$
    – Cosmas Zachos
    Feb 20 at 12:15














  • 1




    $begingroup$
    WP.
    $endgroup$
    – Cosmas Zachos
    Feb 20 at 12:15








1




1




$begingroup$
WP.
$endgroup$
– Cosmas Zachos
Feb 20 at 12:15




$begingroup$
WP.
$endgroup$
– Cosmas Zachos
Feb 20 at 12:15










1 Answer
1






active

oldest

votes


















8












$begingroup$

Hint: Use repeatedly the identity for the 1st derivative
$$ frac{d}{dlambda}e^{that{A}} ~=~ int_0^t!dt_1~ e^{(t-t_1)hat{A}}frac{dhat{A}}{dlambda}e^{t_1hat{A}}~=~ iint_{mathbb{R}^2_+}!dt_1~dt_2~delta(t!-!t_1!-!t_2)~ e^{t_2hat{A}}frac{dhat{A}}{dlambda}e^{t_1hat{A}} , qquad t~in~mathbb{R}_+ .tag{1} $$
For a proof of eq. (1) see e.g. my Phys.SE answer here. Then the 2nd derivative becomes
$$begin{align} frac{1}{2!}frac{d^2}{dlambda^2}e^{that{A}}
&~=~iiint_{mathbb{R}^3_+}!dt_1~dt_2~dt_3~delta(t!-!t_1!-!t_2!-!t_3)~ e^{t_3hat{A}}frac{dhat{A}}{dlambda}e^{t_2hat{A}}frac{dhat{A}}{dlambda}e^{t_1hat{A}} cr
&~+~ iint_{mathbb{R}^2_+}!dt_1~dt_2~delta(t!-!t_1!-!t_2)~ e^{t_2hat{A}}frac{1}{2!}frac{d^2hat{A}}{dlambda^2}e^{t_1hat{A}} , qquad t~in~mathbb{R}_+ ,end{align}tag{2} $$

the 3rd derivative becomes
$$begin{align} frac{1}{3!}frac{d^3}{dlambda^3}e^{that{A}}
&~=~iiiint_{mathbb{R}^4_+}!dt_1~dt_2~dt_3~dt_4~delta(t!-!t_1!-!t_2!-!t_3!-!t_4)~ e^{t_4hat{A}}frac{dhat{A}}{dlambda}e^{t_3hat{A}}frac{dhat{A}}{dlambda}e^{t_2hat{A}}frac{dhat{A}}{dlambda}e^{t_1hat{A}} cr
&~+~iiint_{mathbb{R}^3_+}!dt_1~dt_2~dt_3~delta(t!-!t_1!-!t_2!-!t_3)~ e^{t_3hat{A}}frac{dhat{A}}{dlambda}e^{t_2hat{A}}frac{1}{2!}frac{d^2hat{A}}{dlambda^2}e^{t_1hat{A}} cr
&~+~iiint_{mathbb{R}^3_+}!dt_1~dt_2~dt_3~delta(t!-!t_1!-!t_2!-!t_3)~ e^{t_3hat{A}}frac{1}{2!}frac{d^2hat{A}}{dlambda^2}e^{t_2hat{A}}frac{dhat{A}}{dlambda}e^{t_1hat{A}} cr
&~+~ iint_{mathbb{R}^2_+}!dt_1~dt_2~delta(t!-!t_1!-!t_2)~ e^{t_2hat{A}}frac{1}{3!}frac{d^3hat{A}}{dlambda^3}e^{t_1hat{A}} , qquad t~in~mathbb{R}_+ ,end{align}tag{3} $$

and so forth. The $N$th derivative $$frac{1}{N!}frac{d^N}{dlambda^N}e^{that{A}}~=~sumstackrel{t_{n+1}}{rule{1cm}{.5mm}}fbox{$k_n$}stackrel{t_n}{rule{1cm}{.5mm}}cdots stackrel{t_3}{rule{1cm}{.5mm}}fbox{$k_2$}stackrel{t_2}{rule{1cm}{.5mm}}fbox{$k_1$}stackrel{t_1}{rule{1cm}{.5mm}}tag{N}$$
is a sum of possible Feynman diagrams with Schwinger propagators $$stackrel{t_i}{rule{1cm}{.5mm}}~=~e^{t_ihat{A}}, qquad t_i~in~mathbb{R}_+,tag{P}$$
and 2-vertices $$fbox{$k_i$}~=~frac{1}{k_i!}frac{d^{k_i}hat{A}}{dlambda^{k_i}}, qquad k_i~in~mathbb{N}.tag{V}$$ Each diagram in the sum has weight 1.






share|cite|improve this answer











$endgroup$













  • $begingroup$
    That's horrifying.
    $endgroup$
    – Emilio Pisanty
    Feb 21 at 18:17






  • 1




    $begingroup$
    Indeed a computational nighmare. But there is beauty behind the madness.
    $endgroup$
    – Qmechanic
    Feb 23 at 13:28












  • $begingroup$
    The problem with that is that you need to get through the madness to get to the beauty.
    $endgroup$
    – Emilio Pisanty
    Feb 23 at 13:30










  • $begingroup$
    $uparrow$ Ha-ha.
    $endgroup$
    – Qmechanic
    Feb 23 at 14:20











Your Answer





StackExchange.ifUsing("editor", function () {
return StackExchange.using("mathjaxEditing", function () {
StackExchange.MarkdownEditor.creationCallbacks.add(function (editor, postfix) {
StackExchange.mathjaxEditing.prepareWmdForMathJax(editor, postfix, [["$", "$"], ["\\(","\\)"]]);
});
});
}, "mathjax-editing");

StackExchange.ready(function() {
var channelOptions = {
tags: "".split(" "),
id: "151"
};
initTagRenderer("".split(" "), "".split(" "), channelOptions);

StackExchange.using("externalEditor", function() {
// Have to fire editor after snippets, if snippets enabled
if (StackExchange.settings.snippets.snippetsEnabled) {
StackExchange.using("snippets", function() {
createEditor();
});
}
else {
createEditor();
}
});

function createEditor() {
StackExchange.prepareEditor({
heartbeatType: 'answer',
autoActivateHeartbeat: false,
convertImagesToLinks: false,
noModals: true,
showLowRepImageUploadWarning: true,
reputationToPostImages: null,
bindNavPrevention: true,
postfix: "",
imageUploader: {
brandingHtml: "Powered by u003ca class="icon-imgur-white" href="https://imgur.com/"u003eu003c/au003e",
contentPolicyHtml: "User contributions licensed under u003ca href="https://creativecommons.org/licenses/by-sa/3.0/"u003ecc by-sa 3.0 with attribution requiredu003c/au003e u003ca href="https://stackoverflow.com/legal/content-policy"u003e(content policy)u003c/au003e",
allowUrls: true
},
noCode: true, onDemand: true,
discardSelector: ".discard-answer"
,immediatelyShowMarkdownHelp:true
});


}
});














draft saved

draft discarded


















StackExchange.ready(
function () {
StackExchange.openid.initPostLogin('.new-post-login', 'https%3a%2f%2fphysics.stackexchange.com%2fquestions%2f461860%2fgeneral-derivative-of-the-exponential-operator-w-r-t-a-parameter%23new-answer', 'question_page');
}
);

Post as a guest















Required, but never shown

























1 Answer
1






active

oldest

votes








1 Answer
1






active

oldest

votes









active

oldest

votes






active

oldest

votes









8












$begingroup$

Hint: Use repeatedly the identity for the 1st derivative
$$ frac{d}{dlambda}e^{that{A}} ~=~ int_0^t!dt_1~ e^{(t-t_1)hat{A}}frac{dhat{A}}{dlambda}e^{t_1hat{A}}~=~ iint_{mathbb{R}^2_+}!dt_1~dt_2~delta(t!-!t_1!-!t_2)~ e^{t_2hat{A}}frac{dhat{A}}{dlambda}e^{t_1hat{A}} , qquad t~in~mathbb{R}_+ .tag{1} $$
For a proof of eq. (1) see e.g. my Phys.SE answer here. Then the 2nd derivative becomes
$$begin{align} frac{1}{2!}frac{d^2}{dlambda^2}e^{that{A}}
&~=~iiint_{mathbb{R}^3_+}!dt_1~dt_2~dt_3~delta(t!-!t_1!-!t_2!-!t_3)~ e^{t_3hat{A}}frac{dhat{A}}{dlambda}e^{t_2hat{A}}frac{dhat{A}}{dlambda}e^{t_1hat{A}} cr
&~+~ iint_{mathbb{R}^2_+}!dt_1~dt_2~delta(t!-!t_1!-!t_2)~ e^{t_2hat{A}}frac{1}{2!}frac{d^2hat{A}}{dlambda^2}e^{t_1hat{A}} , qquad t~in~mathbb{R}_+ ,end{align}tag{2} $$

the 3rd derivative becomes
$$begin{align} frac{1}{3!}frac{d^3}{dlambda^3}e^{that{A}}
&~=~iiiint_{mathbb{R}^4_+}!dt_1~dt_2~dt_3~dt_4~delta(t!-!t_1!-!t_2!-!t_3!-!t_4)~ e^{t_4hat{A}}frac{dhat{A}}{dlambda}e^{t_3hat{A}}frac{dhat{A}}{dlambda}e^{t_2hat{A}}frac{dhat{A}}{dlambda}e^{t_1hat{A}} cr
&~+~iiint_{mathbb{R}^3_+}!dt_1~dt_2~dt_3~delta(t!-!t_1!-!t_2!-!t_3)~ e^{t_3hat{A}}frac{dhat{A}}{dlambda}e^{t_2hat{A}}frac{1}{2!}frac{d^2hat{A}}{dlambda^2}e^{t_1hat{A}} cr
&~+~iiint_{mathbb{R}^3_+}!dt_1~dt_2~dt_3~delta(t!-!t_1!-!t_2!-!t_3)~ e^{t_3hat{A}}frac{1}{2!}frac{d^2hat{A}}{dlambda^2}e^{t_2hat{A}}frac{dhat{A}}{dlambda}e^{t_1hat{A}} cr
&~+~ iint_{mathbb{R}^2_+}!dt_1~dt_2~delta(t!-!t_1!-!t_2)~ e^{t_2hat{A}}frac{1}{3!}frac{d^3hat{A}}{dlambda^3}e^{t_1hat{A}} , qquad t~in~mathbb{R}_+ ,end{align}tag{3} $$

and so forth. The $N$th derivative $$frac{1}{N!}frac{d^N}{dlambda^N}e^{that{A}}~=~sumstackrel{t_{n+1}}{rule{1cm}{.5mm}}fbox{$k_n$}stackrel{t_n}{rule{1cm}{.5mm}}cdots stackrel{t_3}{rule{1cm}{.5mm}}fbox{$k_2$}stackrel{t_2}{rule{1cm}{.5mm}}fbox{$k_1$}stackrel{t_1}{rule{1cm}{.5mm}}tag{N}$$
is a sum of possible Feynman diagrams with Schwinger propagators $$stackrel{t_i}{rule{1cm}{.5mm}}~=~e^{t_ihat{A}}, qquad t_i~in~mathbb{R}_+,tag{P}$$
and 2-vertices $$fbox{$k_i$}~=~frac{1}{k_i!}frac{d^{k_i}hat{A}}{dlambda^{k_i}}, qquad k_i~in~mathbb{N}.tag{V}$$ Each diagram in the sum has weight 1.






share|cite|improve this answer











$endgroup$













  • $begingroup$
    That's horrifying.
    $endgroup$
    – Emilio Pisanty
    Feb 21 at 18:17






  • 1




    $begingroup$
    Indeed a computational nighmare. But there is beauty behind the madness.
    $endgroup$
    – Qmechanic
    Feb 23 at 13:28












  • $begingroup$
    The problem with that is that you need to get through the madness to get to the beauty.
    $endgroup$
    – Emilio Pisanty
    Feb 23 at 13:30










  • $begingroup$
    $uparrow$ Ha-ha.
    $endgroup$
    – Qmechanic
    Feb 23 at 14:20
















8












$begingroup$

Hint: Use repeatedly the identity for the 1st derivative
$$ frac{d}{dlambda}e^{that{A}} ~=~ int_0^t!dt_1~ e^{(t-t_1)hat{A}}frac{dhat{A}}{dlambda}e^{t_1hat{A}}~=~ iint_{mathbb{R}^2_+}!dt_1~dt_2~delta(t!-!t_1!-!t_2)~ e^{t_2hat{A}}frac{dhat{A}}{dlambda}e^{t_1hat{A}} , qquad t~in~mathbb{R}_+ .tag{1} $$
For a proof of eq. (1) see e.g. my Phys.SE answer here. Then the 2nd derivative becomes
$$begin{align} frac{1}{2!}frac{d^2}{dlambda^2}e^{that{A}}
&~=~iiint_{mathbb{R}^3_+}!dt_1~dt_2~dt_3~delta(t!-!t_1!-!t_2!-!t_3)~ e^{t_3hat{A}}frac{dhat{A}}{dlambda}e^{t_2hat{A}}frac{dhat{A}}{dlambda}e^{t_1hat{A}} cr
&~+~ iint_{mathbb{R}^2_+}!dt_1~dt_2~delta(t!-!t_1!-!t_2)~ e^{t_2hat{A}}frac{1}{2!}frac{d^2hat{A}}{dlambda^2}e^{t_1hat{A}} , qquad t~in~mathbb{R}_+ ,end{align}tag{2} $$

the 3rd derivative becomes
$$begin{align} frac{1}{3!}frac{d^3}{dlambda^3}e^{that{A}}
&~=~iiiint_{mathbb{R}^4_+}!dt_1~dt_2~dt_3~dt_4~delta(t!-!t_1!-!t_2!-!t_3!-!t_4)~ e^{t_4hat{A}}frac{dhat{A}}{dlambda}e^{t_3hat{A}}frac{dhat{A}}{dlambda}e^{t_2hat{A}}frac{dhat{A}}{dlambda}e^{t_1hat{A}} cr
&~+~iiint_{mathbb{R}^3_+}!dt_1~dt_2~dt_3~delta(t!-!t_1!-!t_2!-!t_3)~ e^{t_3hat{A}}frac{dhat{A}}{dlambda}e^{t_2hat{A}}frac{1}{2!}frac{d^2hat{A}}{dlambda^2}e^{t_1hat{A}} cr
&~+~iiint_{mathbb{R}^3_+}!dt_1~dt_2~dt_3~delta(t!-!t_1!-!t_2!-!t_3)~ e^{t_3hat{A}}frac{1}{2!}frac{d^2hat{A}}{dlambda^2}e^{t_2hat{A}}frac{dhat{A}}{dlambda}e^{t_1hat{A}} cr
&~+~ iint_{mathbb{R}^2_+}!dt_1~dt_2~delta(t!-!t_1!-!t_2)~ e^{t_2hat{A}}frac{1}{3!}frac{d^3hat{A}}{dlambda^3}e^{t_1hat{A}} , qquad t~in~mathbb{R}_+ ,end{align}tag{3} $$

and so forth. The $N$th derivative $$frac{1}{N!}frac{d^N}{dlambda^N}e^{that{A}}~=~sumstackrel{t_{n+1}}{rule{1cm}{.5mm}}fbox{$k_n$}stackrel{t_n}{rule{1cm}{.5mm}}cdots stackrel{t_3}{rule{1cm}{.5mm}}fbox{$k_2$}stackrel{t_2}{rule{1cm}{.5mm}}fbox{$k_1$}stackrel{t_1}{rule{1cm}{.5mm}}tag{N}$$
is a sum of possible Feynman diagrams with Schwinger propagators $$stackrel{t_i}{rule{1cm}{.5mm}}~=~e^{t_ihat{A}}, qquad t_i~in~mathbb{R}_+,tag{P}$$
and 2-vertices $$fbox{$k_i$}~=~frac{1}{k_i!}frac{d^{k_i}hat{A}}{dlambda^{k_i}}, qquad k_i~in~mathbb{N}.tag{V}$$ Each diagram in the sum has weight 1.






share|cite|improve this answer











$endgroup$













  • $begingroup$
    That's horrifying.
    $endgroup$
    – Emilio Pisanty
    Feb 21 at 18:17






  • 1




    $begingroup$
    Indeed a computational nighmare. But there is beauty behind the madness.
    $endgroup$
    – Qmechanic
    Feb 23 at 13:28












  • $begingroup$
    The problem with that is that you need to get through the madness to get to the beauty.
    $endgroup$
    – Emilio Pisanty
    Feb 23 at 13:30










  • $begingroup$
    $uparrow$ Ha-ha.
    $endgroup$
    – Qmechanic
    Feb 23 at 14:20














8












8








8





$begingroup$

Hint: Use repeatedly the identity for the 1st derivative
$$ frac{d}{dlambda}e^{that{A}} ~=~ int_0^t!dt_1~ e^{(t-t_1)hat{A}}frac{dhat{A}}{dlambda}e^{t_1hat{A}}~=~ iint_{mathbb{R}^2_+}!dt_1~dt_2~delta(t!-!t_1!-!t_2)~ e^{t_2hat{A}}frac{dhat{A}}{dlambda}e^{t_1hat{A}} , qquad t~in~mathbb{R}_+ .tag{1} $$
For a proof of eq. (1) see e.g. my Phys.SE answer here. Then the 2nd derivative becomes
$$begin{align} frac{1}{2!}frac{d^2}{dlambda^2}e^{that{A}}
&~=~iiint_{mathbb{R}^3_+}!dt_1~dt_2~dt_3~delta(t!-!t_1!-!t_2!-!t_3)~ e^{t_3hat{A}}frac{dhat{A}}{dlambda}e^{t_2hat{A}}frac{dhat{A}}{dlambda}e^{t_1hat{A}} cr
&~+~ iint_{mathbb{R}^2_+}!dt_1~dt_2~delta(t!-!t_1!-!t_2)~ e^{t_2hat{A}}frac{1}{2!}frac{d^2hat{A}}{dlambda^2}e^{t_1hat{A}} , qquad t~in~mathbb{R}_+ ,end{align}tag{2} $$

the 3rd derivative becomes
$$begin{align} frac{1}{3!}frac{d^3}{dlambda^3}e^{that{A}}
&~=~iiiint_{mathbb{R}^4_+}!dt_1~dt_2~dt_3~dt_4~delta(t!-!t_1!-!t_2!-!t_3!-!t_4)~ e^{t_4hat{A}}frac{dhat{A}}{dlambda}e^{t_3hat{A}}frac{dhat{A}}{dlambda}e^{t_2hat{A}}frac{dhat{A}}{dlambda}e^{t_1hat{A}} cr
&~+~iiint_{mathbb{R}^3_+}!dt_1~dt_2~dt_3~delta(t!-!t_1!-!t_2!-!t_3)~ e^{t_3hat{A}}frac{dhat{A}}{dlambda}e^{t_2hat{A}}frac{1}{2!}frac{d^2hat{A}}{dlambda^2}e^{t_1hat{A}} cr
&~+~iiint_{mathbb{R}^3_+}!dt_1~dt_2~dt_3~delta(t!-!t_1!-!t_2!-!t_3)~ e^{t_3hat{A}}frac{1}{2!}frac{d^2hat{A}}{dlambda^2}e^{t_2hat{A}}frac{dhat{A}}{dlambda}e^{t_1hat{A}} cr
&~+~ iint_{mathbb{R}^2_+}!dt_1~dt_2~delta(t!-!t_1!-!t_2)~ e^{t_2hat{A}}frac{1}{3!}frac{d^3hat{A}}{dlambda^3}e^{t_1hat{A}} , qquad t~in~mathbb{R}_+ ,end{align}tag{3} $$

and so forth. The $N$th derivative $$frac{1}{N!}frac{d^N}{dlambda^N}e^{that{A}}~=~sumstackrel{t_{n+1}}{rule{1cm}{.5mm}}fbox{$k_n$}stackrel{t_n}{rule{1cm}{.5mm}}cdots stackrel{t_3}{rule{1cm}{.5mm}}fbox{$k_2$}stackrel{t_2}{rule{1cm}{.5mm}}fbox{$k_1$}stackrel{t_1}{rule{1cm}{.5mm}}tag{N}$$
is a sum of possible Feynman diagrams with Schwinger propagators $$stackrel{t_i}{rule{1cm}{.5mm}}~=~e^{t_ihat{A}}, qquad t_i~in~mathbb{R}_+,tag{P}$$
and 2-vertices $$fbox{$k_i$}~=~frac{1}{k_i!}frac{d^{k_i}hat{A}}{dlambda^{k_i}}, qquad k_i~in~mathbb{N}.tag{V}$$ Each diagram in the sum has weight 1.






share|cite|improve this answer











$endgroup$



Hint: Use repeatedly the identity for the 1st derivative
$$ frac{d}{dlambda}e^{that{A}} ~=~ int_0^t!dt_1~ e^{(t-t_1)hat{A}}frac{dhat{A}}{dlambda}e^{t_1hat{A}}~=~ iint_{mathbb{R}^2_+}!dt_1~dt_2~delta(t!-!t_1!-!t_2)~ e^{t_2hat{A}}frac{dhat{A}}{dlambda}e^{t_1hat{A}} , qquad t~in~mathbb{R}_+ .tag{1} $$
For a proof of eq. (1) see e.g. my Phys.SE answer here. Then the 2nd derivative becomes
$$begin{align} frac{1}{2!}frac{d^2}{dlambda^2}e^{that{A}}
&~=~iiint_{mathbb{R}^3_+}!dt_1~dt_2~dt_3~delta(t!-!t_1!-!t_2!-!t_3)~ e^{t_3hat{A}}frac{dhat{A}}{dlambda}e^{t_2hat{A}}frac{dhat{A}}{dlambda}e^{t_1hat{A}} cr
&~+~ iint_{mathbb{R}^2_+}!dt_1~dt_2~delta(t!-!t_1!-!t_2)~ e^{t_2hat{A}}frac{1}{2!}frac{d^2hat{A}}{dlambda^2}e^{t_1hat{A}} , qquad t~in~mathbb{R}_+ ,end{align}tag{2} $$

the 3rd derivative becomes
$$begin{align} frac{1}{3!}frac{d^3}{dlambda^3}e^{that{A}}
&~=~iiiint_{mathbb{R}^4_+}!dt_1~dt_2~dt_3~dt_4~delta(t!-!t_1!-!t_2!-!t_3!-!t_4)~ e^{t_4hat{A}}frac{dhat{A}}{dlambda}e^{t_3hat{A}}frac{dhat{A}}{dlambda}e^{t_2hat{A}}frac{dhat{A}}{dlambda}e^{t_1hat{A}} cr
&~+~iiint_{mathbb{R}^3_+}!dt_1~dt_2~dt_3~delta(t!-!t_1!-!t_2!-!t_3)~ e^{t_3hat{A}}frac{dhat{A}}{dlambda}e^{t_2hat{A}}frac{1}{2!}frac{d^2hat{A}}{dlambda^2}e^{t_1hat{A}} cr
&~+~iiint_{mathbb{R}^3_+}!dt_1~dt_2~dt_3~delta(t!-!t_1!-!t_2!-!t_3)~ e^{t_3hat{A}}frac{1}{2!}frac{d^2hat{A}}{dlambda^2}e^{t_2hat{A}}frac{dhat{A}}{dlambda}e^{t_1hat{A}} cr
&~+~ iint_{mathbb{R}^2_+}!dt_1~dt_2~delta(t!-!t_1!-!t_2)~ e^{t_2hat{A}}frac{1}{3!}frac{d^3hat{A}}{dlambda^3}e^{t_1hat{A}} , qquad t~in~mathbb{R}_+ ,end{align}tag{3} $$

and so forth. The $N$th derivative $$frac{1}{N!}frac{d^N}{dlambda^N}e^{that{A}}~=~sumstackrel{t_{n+1}}{rule{1cm}{.5mm}}fbox{$k_n$}stackrel{t_n}{rule{1cm}{.5mm}}cdots stackrel{t_3}{rule{1cm}{.5mm}}fbox{$k_2$}stackrel{t_2}{rule{1cm}{.5mm}}fbox{$k_1$}stackrel{t_1}{rule{1cm}{.5mm}}tag{N}$$
is a sum of possible Feynman diagrams with Schwinger propagators $$stackrel{t_i}{rule{1cm}{.5mm}}~=~e^{t_ihat{A}}, qquad t_i~in~mathbb{R}_+,tag{P}$$
and 2-vertices $$fbox{$k_i$}~=~frac{1}{k_i!}frac{d^{k_i}hat{A}}{dlambda^{k_i}}, qquad k_i~in~mathbb{N}.tag{V}$$ Each diagram in the sum has weight 1.







share|cite|improve this answer














share|cite|improve this answer



share|cite|improve this answer








edited Feb 23 at 14:21

























answered Feb 20 at 11:16









QmechanicQmechanic

105k121901203




105k121901203












  • $begingroup$
    That's horrifying.
    $endgroup$
    – Emilio Pisanty
    Feb 21 at 18:17






  • 1




    $begingroup$
    Indeed a computational nighmare. But there is beauty behind the madness.
    $endgroup$
    – Qmechanic
    Feb 23 at 13:28












  • $begingroup$
    The problem with that is that you need to get through the madness to get to the beauty.
    $endgroup$
    – Emilio Pisanty
    Feb 23 at 13:30










  • $begingroup$
    $uparrow$ Ha-ha.
    $endgroup$
    – Qmechanic
    Feb 23 at 14:20


















  • $begingroup$
    That's horrifying.
    $endgroup$
    – Emilio Pisanty
    Feb 21 at 18:17






  • 1




    $begingroup$
    Indeed a computational nighmare. But there is beauty behind the madness.
    $endgroup$
    – Qmechanic
    Feb 23 at 13:28












  • $begingroup$
    The problem with that is that you need to get through the madness to get to the beauty.
    $endgroup$
    – Emilio Pisanty
    Feb 23 at 13:30










  • $begingroup$
    $uparrow$ Ha-ha.
    $endgroup$
    – Qmechanic
    Feb 23 at 14:20
















$begingroup$
That's horrifying.
$endgroup$
– Emilio Pisanty
Feb 21 at 18:17




$begingroup$
That's horrifying.
$endgroup$
– Emilio Pisanty
Feb 21 at 18:17




1




1




$begingroup$
Indeed a computational nighmare. But there is beauty behind the madness.
$endgroup$
– Qmechanic
Feb 23 at 13:28






$begingroup$
Indeed a computational nighmare. But there is beauty behind the madness.
$endgroup$
– Qmechanic
Feb 23 at 13:28














$begingroup$
The problem with that is that you need to get through the madness to get to the beauty.
$endgroup$
– Emilio Pisanty
Feb 23 at 13:30




$begingroup$
The problem with that is that you need to get through the madness to get to the beauty.
$endgroup$
– Emilio Pisanty
Feb 23 at 13:30












$begingroup$
$uparrow$ Ha-ha.
$endgroup$
– Qmechanic
Feb 23 at 14:20




$begingroup$
$uparrow$ Ha-ha.
$endgroup$
– Qmechanic
Feb 23 at 14:20


















draft saved

draft discarded




















































Thanks for contributing an answer to Physics Stack Exchange!


  • Please be sure to answer the question. Provide details and share your research!

But avoid



  • Asking for help, clarification, or responding to other answers.

  • Making statements based on opinion; back them up with references or personal experience.


Use MathJax to format equations. MathJax reference.


To learn more, see our tips on writing great answers.




draft saved


draft discarded














StackExchange.ready(
function () {
StackExchange.openid.initPostLogin('.new-post-login', 'https%3a%2f%2fphysics.stackexchange.com%2fquestions%2f461860%2fgeneral-derivative-of-the-exponential-operator-w-r-t-a-parameter%23new-answer', 'question_page');
}
);

Post as a guest















Required, but never shown





















































Required, but never shown














Required, but never shown












Required, but never shown







Required, but never shown

































Required, but never shown














Required, but never shown












Required, but never shown







Required, but never shown







Popular posts from this blog

Biblatex bibliography style without URLs when DOI exists (in Overleaf with Zotero bibliography)

ComboBox Display Member on multiple fields

Is it possible to collect Nectar points via Trainline?